Frage:
Wie finde ich alle geraden Momente (und ungeraden Momente) für $ f_X (x) = \ frac {1} {2} e ^ {- | x |} $?
Ron Snow
2019-09-17 02:49:51 UTC
view on stackexchange narkive permalink

Ich wurde gebeten, eine Formel für alle geraden Momente der Form $ E (X ^ {2n}) $ span> und alle ungeraden Momente der Form $ E (X ^ {2n + 1}) $ span> mit einem mgf.Können Sie mir helfen, die ausgeglichenen Momente zu finden?Ich werde versuchen, die seltsamen Momente mit ähnlichen Techniken zu lösen.

Sei $ X $ span> eine kontinuierliche Zufallsvariable mit der Dichte $ f_X (x) = \ frac {1}{2} e ^ {- | x |} $ span>, wobei $ - \ infty < x < \ infty $ span>.

Meine Arbeit: Ich konnte die folgende momentgenerierende Funktion erhalten: $ M_X (t) = (1-t ^ 2) ^ {- 1}. $ Ich bin mir jedoch nicht sicher, wohin ich von hier aus gehen soll.Vielen Dank für Ihre Hilfe.

Hinweis: Wenn Sie $ M_X (t) $ als Potenzreihe in $ t $ ausdrücken können (dh etwas, das wie $ 1 + m_1t + m_2t ^ 2 + m_3 t ^ 3 + \ cdots $ aussieht), können Sie finden$ E [X ^ n] $ in Bezug auf $ m_n $
Ich glaube ich sehe was du sagst.Da $ M_X (t) = (1-t ^ 2) ^ {- 1} $, ähnelt es leicht einer geometrischen Reihe: $ (1-t ^ 2) ^ {- 1} = 1 + t ^ 2 + (t ^ 2) ^ 2 + (t ^ 2 ^) 3+ \ cdots $.Dies kann nur passieren, wenn $ | t ^ 2 |<1, $ was erfüllt ist
Über merkwürdige Momente: Haben Sie die Verteilung aufgezeichnet?Was wäre der Mittelwert?der Versatz?Können Sie etwas über merkwürdige Momente für symetrische Verteilungen schließen?
Ich habe die Verteilung geplant.Der Mittelwert scheint 0 zu sein und die Darstellung ist symmetrisch.Ich habe diesen Link gefunden, der darauf schließen lässt, dass ungerade Momente symmetrischer Verteilungen 0 sind. Wie kann ich dieses Ergebnis mit mgfs anzeigen?https://math.stackexchange.com/questions/72451/odd-order-moments-of-a-symmetrical-distribution
Haben Sie keinen Text oder keine Theorie, die Sie bei Ihren Übungen unterstützen könnten?Ich glaube, dass Sie diese Frage zuerst wirklich selbst untersuchen sollten.Beginnen Sie möglicherweise hier https://stats.stackexchange.com/questions/238776/
Sie haben $ f x (x) $, wo Sie $ f X (x) benötigen.\ qquad $
Der Text, den ich verwende, ist Casella & Berger.
Es enthält nicht die Erklärung, was Sie mit der Momenterzeugungsfunktion tun sollen, um die Momente zu erhalten?
Es gibt nur die folgende Eigenschaft: $ E (X ^ n) = M_X ^ {(n)} (0) = \ frac {d ^ n} {dt ^ n} M_X (t) | _ {t = 0} $.Ich hätte also $ E (X ^ {2n}) = M_X ^ {(2n)} (0) = \ frac {d ^ {2n}} {dt ^ {2n}} M_X (t) | _ {t= 0} $, die ich nicht auf eine Formel verallgemeinern kann.
Also habe ich die erste, zweite, dritte und vierte Ableitung berechnet.Ich habe $ E (X ^ 1) = 0, E (X ^ 2) = 2, E (X ^ 3) = 0, $ und $ E (X ^ 4) = 12 $.Diese Ableitungen sind zu diesem Zeitpunkt ziemlich lang zu berechnen, daher frage ich mich, ob es einen einfacheren Weg gibt, eine Formel für die Evens zu erhalten.Bei der zweiten und vierten Ableitung scheint die Formel für $ E (X ^ {2n}) $ immer noch unklar zu sein.
Fünf antworten:
probabilityislogic
2019-09-17 06:13:19 UTC
view on stackexchange narkive permalink

Eine Möglichkeit, Ihren mgf-Ansatz für das Problem zu vereinfachen, ist die Verwendung der Potenzreihe

$$ (1-t ^ 2) ^ {- 1} = \ sum_ {j = 0} ^ {\ infty} t ^ {2j} $$ span >

Das wiederholte Differenzieren der rhs ist viel einfacher als das Differenzieren der lhs. (Beachten Sie, dass dies nur für $ | t | <1 $ span> gilt, aber da Sie bei $ t = 0 $ span differenzieren > es funktioniert immer noch). Sie sollten sehen, dass es nur eine Fakultät sein wird. Es ist jedoch wahrscheinlich noch einfacher, die Erwartung direkt zu bewerten

$$ E (X ^ {2n}) = \ int _ {- \ infty} ^ {\ infty} \ frac {x ^ {2n}} {2} \ exp (- | x |) dx $$ span>

Das Ergebnis sollte eine Gammafunktion sein (dh faktoriell). Dies wird auch aus der mgf-Potenzreihe deutlich.

update In Bezug auf den mgf-Ansatz müssen wir zur Bewertung von $ E (X ^ {2n}) $ span> die rhs $ 2n $ unterscheiden span> mal. Das folgende Ergebnis ist nützlich

$$ \ frac {\ partielle ^ k x ^ {r}} {\ partielle x ^ k} = \ left ( \ begin {matrix} \ frac {r!} {(r-k)!} x ^ {r-k} & r = k, k + 1, \ dots \\ 0 & r = 1, \ dots, k-1 \ end {matrix} \Recht) $$ span>

Wenn Sie dies nun auf den Begriff $ t ^ {2j} $ span> anwenden und die Ableitung "2n-th" nehmen, haben wir $ r = 2j $ span> und $ k = 2n $ span> und $ x = t $ . Dann bekommen wir $$ \ frac {\ partiell ^ {2n} t ^ {2j}} {\ partiell t ^ {2n}} = \ left ( \ begin {matrix} \ frac {(2j)!} {(2j-2n)!} t ^ {2j-2n} & 2j = 2n, 2n + 2,2n + 4,2n + 6, \ dots \\ 0 & 2j = 0,2, \ Punkte, 2n-2 \ end {matrix} \Recht) $$ span>

Dies bedeutet, wenn wir alle Begriffe addieren, können wir dies als schreiben $$ \ sum_ {j = 0} ^ {\ infty} \ frac {\ partiell ^ {2n} t ^ {2j}} {\ partiell t ^ {2n}} = 0 + \ Punkte + 0 + (2n)! + t ^ 2 \ frac {(2n + 2)!} {2!} + t ^ 4 \ frac {(2n + 4)!} {4!} + \ Punkte $ $ span>

nur das einzelne tertm $ (2n)! $ span> ist nicht $ 0 $ span> und auch kein Vielfachesvon $ t $ span>.Wenn Sie also $ t = 0 $ span> setzen, bleibt nur dieser Begriff übrig.

Vielen Dank, dass Sie beide Möglichkeiten zur Lösung dieses Problems angegeben haben.Ich denke, ich würde den letzteren Ansatz verwenden, wenn ich die Wahl hätte - es scheint direkter zu sein.
Meine zweite Ableitung der rhs ist $ \ Sigma ^ \ infty _ {j = 0} [2j (2j-1) t ^ {2j-2}] $, aber wenn dies bei $ t = 0 $ ausgewertet wird, ist esgleich Null.Wo vermassle ich es?
Es wird immer einen Term geben, der eine Potenz von Null hat, so dass Sie bei $ t = 0 $ $ t ^ 0 = 0 ^ 0 = 1 $ erhalten und das Ergebnis der Koeffizient für diese Potenz ist.
Aha. Vielen Dank.
Die Updates zu diesem Beitrag werden sehr geschätzt.Vielen Dank für die Erklärung.
Ben
2019-09-17 03:45:43 UTC
view on stackexchange narkive permalink

Hint: Dies ist ein Beispiel für eine Wahrscheinlichkeitsdichtefunktion, die symmetrisch um Null ist:

$$ f_X (0 + x) = f_X (0-x) \ quad \ quad \ quad \ text {für alle} x \ in \ mathbb {R}. $$ span>

Visuell bedeutet dies, dass die Verteilung um die Nulllinie herum reflektiert wird und auf beiden Seiten gleich ist.Sehen Sie, ob Sie diese Eigenschaft verwenden können, um herauszufinden (und dann zu beweisen), was die ungeraden Momente wären.Die geraden Momente sind etwas kniffliger, aber prüfen Sie, ob Sie diese Eigenschaft verwenden können, um diese Momente auf eine einfachere Form zu reduzieren.

Vielen Dank für diesen Hinweis.Ich sehe, wie ungerade Momente um eine symmetrische Verteilung gleich 0 wären, aber wie kann ich dieses Ergebnis von meinem MGF erhalten?
Ich denke, die beste Methode ist es, es als Potenzreihe zu erweitern, wie die ausgezeichnete Antwort von [Wahrscheinlichkeitslogik] (https://stats.stackexchange.com/users/2392/probabilityislogic) zeigt.
Dilip Sarwate
2019-09-17 07:06:29 UTC
view on stackexchange narkive permalink

Da das OP Schwierigkeiten mit den verschiedenen Hinweisen in den Kommentaren und den anderen Antworten zu haben scheint, ist hier eine heuristische Methode, die in diesem Fall die richtige Antwort liefert. \ begin {align} E [\ exp (tX)] & = E \ left [1 + tX + \ frac {(tX) ^ 2} {2!} + \ Frac {(tX) ^ 3} {3!} + \ Cdots \ right ] \\ & = 1 + tE [X] + \ frac {t ^ 2} {2!} E [X ^ 2] + \ frac {t ^ 3} {3!} E [X ^ 3] + \ cdots \ tag { 1} \\ \ end {align} span> und wenn wir eine Potenzreihe für $ E [\ exp (tX)] = M_X (t) $ span> haben, die wir geschafft haben Finden Sie durch Haken oder Gauner und ohne die oben angezeigte Gleichung zu betrachten, da dies nur dazu dient, uns zu verwirren, können wir nur den Koeffizienten von betrachten "math-container"> $ t ^ n $ span> in der Potenzreihe, die wir haben und sagen "Hey Ma! Ich denke, dass der Koeffizient von $ t ^ n $ span> ist nur $ \ frac {1} {n!} E [X ^ n] $ span> und so kann ich $ E [X ^ n] $ span> durch Multiplizieren des Koeffizienten, den ich bereits habe, mit $ n! $ span>.

Für den speziellen Fall des OP hat er bereits festgestellt, dass $$ M_X (t) = \ frac {1} {1-t ^ 2} = 1 + t ^ 2 + t ^ 4 + \ cdots + t ^ {2n} + \ cdots \ tag {2} $$ span> (das ist der Teil "by hook or by crook") und so kann er $ (1) $ span> und $ (2) vergleichen ) $ span> um herauszufinden, was die Momente sind. Ich überlasse es dem OP, Ma mitzuteilen, dass $ E [X ^ n] = 0 $ span>, wenn $ n $ span> ist ungerade.Ob er Ma sagen möchte, was $ E [X ^ n] $ span> für gerade $ n $ span> isteine Sache, die er entscheiden muss.Der $ E [X ^ 4] = 12 $ span>, den das OP (in einem Kommentar) berechnet hat, klingt für mich nicht richtig (ich denke, das sollte es sein $ E [X ^ 4] = 4! = 24 $ span>), aber wer soll ich in die heilige Beziehung zwischen Mutter und Kind eingreifen?

Vielen Dank für Ihren unterhaltsamen Beitrag.Ich verstehe, was du sagst.Ich bekomme also $ E [X ^ n] = n! $, Da wir $ n! $ Mit dem Koeffizienten jedes $ t ^ n $ multiplizieren, wobei $ n $ gerade ist.Jetzt verstehe ich, wie Sie sowohl $ E [e ^ {tX}] $ als auch $ M_X (t) $ erweitert haben, aber wie können Sie die Koeffizienten der beiden Erweiterungen miteinander verwenden?Ich würde gerne das große Ganze verstehen.
whuber
2019-09-17 17:42:00 UTC
view on stackexchange narkive permalink

Wenn Sie keine unnötigen Berechnungen durchführen möchten, können Sie mit it Ihre Verteilung als gleiche Mischung eines Exponentials und seines Negativs anzeigen:

$$ \ frac {1} {2} e ^ {- | x |} = \ frac {1} {2} e ^ {- x} \, \ mathcal {I} (x \ gt 0) + \ frac {1} {2} e ^ {x} \, \ mathcal {I} (x \ lt 0). $$ span>

Weil $ ((- 1) ^ n + (1) ^ n) / 2 $ span> entweder $ - ist 1 + 1 = 0 $ span> oder $ (1 + 1) / 2 = 2/2 = 1 $ span> als $ n $ span> ist ungerade bzw. gerade, die ungeraden Momente Ihrer Verteilung sind Null und die geraden Momente sind die gleichen wie die des Exponentials. Aber per Definition die geraden Exponentielle Momente sind

$$ \ mu_ {2k} = \ int_0 ^ \ infty x ^ {2k} e ^ {- x} \ mathrm {d} x = \ Gamma (2k + 1 ) = (2k)! $$ span>

Da dies keine schwierigere Berechnung als $ 1 + 1 = 2, $ span> erfordert, ist eine einfachere Lösung kaum vorstellbar.


Wenn Sie stattdessen den momentgenerierenden Ansatz verfolgen möchten, stellt zunächst fest, dass die mgf der Exponentialverteilung $$ \ phi (t) = \ int_0 ^ \ infty e ist ^ {tx} e ^ {- x} \ mathrm {d} x = \ int_0 ^ \ infty e ^ {- (1-t) x} \ mathrm {d} x = \ frac {1} {1-t} . $$ span> Somit ist die mgf dieser Mischung

$$ (\ phi (t) + \ phi (-t)) / 2 = \ frac {1} {2} \ left (\ frac {1} {1 -t} + \ frac {1} {1 - (- t)} \ right) = \ frac {1} {1-t ^ 2}. $$ span>

Dies ist eine Analyse in der Nähe von $ 0 $ span> und entspricht daher der Potenzreihenerweiterung, die durch den Binomialsatz als

angegeben wird

$$ \ frac {1} {1-t ^ 2} = (1 + (-t ^ 2)) ^ {- 1} = \ sum_ {k = 0 } ^ \ infty \ binom {-1} {k} (-t ^ 2) ^ k = \ sum_ {k = 0} ^ \ infty t ^ {2k} = \ sum_ {k = 0} ^ \ infty \ color {rot} {(2k)!} \, \ color {grau} {\ frac {t ^ {2k}} {(2k)!}}, $$ span>

von dem Sie die Momente als Koeffizienten von $ t ^ n / n !: $ span> ablesen können, sehen wir wieder, dass sie für die ungeraden Momente und $ (2k)! $ span> für gerade Momente.

Vielen Dank, dass Sie sowohl den einfachsten als auch den mgf-Weg teilen.
Sextus Empiricus
2019-09-17 19:00:56 UTC
view on stackexchange narkive permalink

Also habe ich die erste, zweite, dritte und vierte Ableitung berechnet. Ich habe $ E (X ^ 1) = 0 $ span>, $ E (X ^ 2) = 2 $ span >, $ E (X ^ 3) = 0 $ span> und $ E (X ^ 4) = 12 $ span>. Diese Ableitungen sind zu diesem Zeitpunkt ziemlich lang zu berechnen, daher frage ich mich, ob es einen einfacheren Weg gibt, eine Formel für die Evens zu erhalten.

Sie können die Taylor-Serienerweiterung verwenden:

$$ \ frac {1} {1-t ^ 2} = \ sum_ {k = 0} ^ \ infty t ^ {2k} $$ span>

Dies ist jedoch eine zirkuläre Argumentation, da die Erweiterung der Taylor-Reihe selbst durch Berechnung der Ableitungen abgeleitet wird. In diesem Fall können Sie genauso gut direkt eine Formel für die Momente höherer Ordnung der Laplace-Verteilung nachschlagen.


Möglicherweise stellen Sie fest, dass die Taylor-Reihenerweiterung indirekt erfolgt - ohne $ f (x) = \ sum_ {n = 0} ^ \ infty f ^ {(n)} / n! t ^ k $ span> - Verwenden Sie stattdessen die Formel für eine geometrische Reihe.

Sie können die Ableitungen jedoch auch manuell ableiten (dh einfache Berechnung mit Kettenregel und Produktregel). Wenn Sie sich das Muster der Begriffe ansehen, werden Sie feststellen, dass viele der Begriffe Null und a werden es entsteht ein regelmäßiges Muster.

Angenommen, wir ersetzen $ u = t ^ 2 $ span>, dann sieht die Ableitung einfacher aus:

$$ \ frac {\ text {d} ^ n} {\ text {d} u ^ n} \ frac {1} {(1-u)} = \ frac {n!} {(1-u) ^ n} $$ span>

Verwenden Sie nun die Formel Faà di Bruno (Kettenregel, die dann mehrmals angewendet wird):

$$ \ frac {\ text {d} ^ n} {\ text {d} t ^ n} \ frac {1} {(1-u)} = \ sum_ {k = 1} ^ n \ frac {k!} {(1-u) ^ k} \ cdot B_ {n, k} (2t, 2,0, ..., 0) $$ span>

wobei $ B_ {n, k} $ span> sich auf Bell-Polynome bezieht. Die meisten Begriffe sind Null und Sie erhalten

$$ \ frac {\ text {d} ^ {2n}} {\ text {d} t ^ {2n}} \ frac {1} {(1-t^ 2)} = \ sum_ {k = 0} ^ n c_ {nk} \ frac {t ^ {2k}} {(1-t ^ 2) ^ {1 + n + k}} $$ span>

mit

$$ c_ {nk} = 2 ^ {2k} \ frac {(2n)!\ cdot (n + k)!} {(n-k)!\ cdot (2k)!} $$ span>

und für den Wert bei $ t = 0 $ span> haben Sie

$$ \ frac {\ text {d} ^ {2n}} {\ text {d} t ^ {2n}} \ frac {1} {(1-t^ 2)} = c_ {n0} = (2n)!$$ span>

Obwohl ich mit der Formel von Faa di Bruno nicht vertraut bin, gefällt mir die Struktur Ihrer Antwort.Vielen Dank für Ihre Ergänzung!
Wenn Sie einmal differenzieren, erhalten Sie $ 2t \ cdot \ frac {1} {(1-t ^ 2)} $ zweimal $ 2 \ cdot \ frac {1} {(1-t ^ 2) ^ 2} + 2t \cdot4t \ cdot \ frac {1} {(1-t ^ 2) ^ 3} $ und so weiter.Es sind Kombinationen von Ableitungen höherer Ordnung von $ f (u) = 1 / (1-u) $ und $ g (t) $, die aufgrund wiederholter Differenzierung (Anwendung von Ketten- und Produktregeln, die diese mehrfachen Begriffe erzeugen) auftreten.Die Formel von Faa di Bruno verfolgt diese Begriffe.In diesem Fall wird es relativ einfach, die Begriffe im Auge zu behalten, da die Ableitungen von $ t ^ 2 $ irgendwann enden.Natürlich sind andere Möglichkeiten, die Serie abzuleiten, einfacher, aber diese ist auch cool.
Das ist ein wirklich interessantes Ergebnis.Vielen Dank für das Teilen.
Es ist tatsächlich eine umständliche Methode, $! N ((1-t) ^ {- n} - (-1-t) ^ {- n}) $ zu berechnen


Diese Fragen und Antworten wurden automatisch aus der englischen Sprache übersetzt.Der ursprüngliche Inhalt ist auf stackexchange verfügbar. Wir danken ihm für die cc by-sa 4.0-Lizenz, unter der er vertrieben wird.
Loading...